Intervallerhaltende Transformationen sind in der speziellen Relativitätstheorie linear

Bei fast allen mir bekannten Beweisen der Lorentz-Transformationen geht man davon aus, dass die erforderlichen Transformationen linear sind. Ich frage mich, ob es eine Möglichkeit gibt, die Linearität zu beweisen:

Beweisen Sie, dass jede Raumzeittransformation ( j 0 , j 1 , j 2 , j 3 ) ( x 0 , x 1 , x 2 , x 3 ) das Intervalle bewahrt, das heißt, so dass

( d j 0 ) 2 ( d j 1 ) 2 ( d j 2 ) 2 ( d j 3 ) 2 = ( d x 0 ) 2 ( d x 1 ) 2 ( d x 2 ) 2 ( d x 3 ) 2

linear ist (unter der Annahme, dass die Ursprünge beider Koordinaten zusammenfallen). Das heißt, zeigen Sie das j ich x j = L j ich während der gesamten Raumzeit konstant ist (das heißt, zeigen Sie, dass L j ich x k = 0 ).

Bisher konnte ich nur das beweisen g ich j L p ich L q j = g p q (wo g ich j ist der metrische Tensor der speziellen Relativitätstheorie) und das L j ich x k = L k ich x j . Irgendwelche weiteren Ideen?

Für die Gruppe der längenerhaltenden Transformationen des 3D-Raums ist das übliche Argument für Linearität, dass sie Dreiecke bewahren. Da die Vektorsumme (Differenz) durch Dreiecke gegeben ist, beweisen die Eindeutigkeitssätze für Dreiecke die Linearität. Ein ähnliches Argument könnte für die Poincaré-Gruppe funktionieren.
@Greg: sehr schönes Argument, aber es streng zu machen, würde einiges an Arbeit erfordern (je nachdem, was als bekannt über die axiomatische Ebenengeometrie und die Art und Weise, wie sie mit der üblichen Vektoranalyse verbunden ist, angenommen werden kann).
Ich glaube nicht, dass Beckos Vermutung streng formuliert ist, und wenn es so wäre, wäre sie meiner Meinung nach falsch. Wir lassen die Annahme von Homogenität und Isotropie weg, aber es ist nicht klar, welche anderen Annahmen wir machen . Wenn die Behauptung in irgendeiner sinnvollen Weise gelten soll, dann muss sie für Raumzeiten gelten, die inhomogen und/oder anisotrop sind. In welcher Theorie operieren wir dann? Ist es ein Verteiler mit einer bestimmten Art von Verbindung? Lassen wir Torsion zu? In einem solchen verallgemeinerten Kontext erwarten wir nicht einmal, dass wir die gesamte Mannigfaltigkeit mit einem einzigen Koordinatendiagramm abdecken können.
Becko, es hört sich so an, als ob Sie nach einem Beweis fragen, dass die Lorentz-Transformation in gekrümmter Raumzeit linear ist, wo Homogenität und Isotropie versagen. Aber in einer gekrümmten Raumzeit gibt es nicht einmal eine Vorstellung von einer globalen Lorentz-Transformation; es existiert nur in einer infinitesimalen Nachbarschaft in der Raumzeit. Beschränkt man sich auf eine infinitesimale Nachbarschaft, dann geht man im Wesentlichen von Homogenität und Isotropie aus, da die Krümmung auf kleinen Skalen nicht mehr nachweisbar ist. Das besagt im Wesentlichen das Äquivalenzprinzip.
@Ben Crowell: Während ich Beckos Frage lese, bittet Becko um einen Beweis, dass eine lokale Koordinatentransformation zwischen zwei lokalen Koordinatensystemen (auf a 3 + 1 dimensionale Lorentz-Mannigfaltigkeit) muss affin sein, wenn die Metrik g μ v in beiden lokalen Koordinatensystemen haben zufällig die Minkowski-Form η μ v = d ich a g ( 1 , + 1 , + 1 , + 1 ) .
@Qmechanic: OK, vielleicht fehlt mir nur etwas, aber das ergibt für mich immer noch keinen Sinn. Wenn angenommen wird, dass die Raumzeit flach ist, ist das dann nicht eine stärkere Annahme als die Annahme von Homogenität und Isotropie?
Kommentar zur Frage (v1): Wenn die beiden Koordinatenfelder vollständige Vektorräume sind (im Gegensatz zu nur echten Teilmengen davon) und wenn die Signatur euklidisch ist, dann wird die Antwort durch das Mazur-Ulam-Theorem gegeben, vgl. en.wikipedia.org/wiki/Mazur%E2%80%93Ulam_theorem
@Ben Crowell: Ich glaube, ich habe mich geirrt, es so auszudrücken. Die Raumzeit ist gemäß der Minkowski-Metrik homogen, isotrop und flach. Was ich nicht annehmen möchte, sind irgendwelche Eigenschaften von Homogenität oder Isotropie in der Koordinatentransformation { j ich } { x ich } . Ich möchte nur annehmen, dass das Intervall erhalten bleibt, und damit die Linearität beweisen. Ich hoffe es ist jetzt klarer.

Antworten (8)

Im Nachhinein hier ein kurzer Beweis.

Die Metrik g μ v ist die flache konstante Metrik η μ v in beiden Koordinatensystemen. Daher die entsprechenden (eindeutig definierten) Levi-Civita Christoffel-Symbole

Γ μ v λ   =   0

sind in beiden Koordinatensystemen Null. Es ist bekannt, dass sich das Christoffel-Symbol bei einer lokalen Koordinatentransformation nicht als Tensor transformiert x μ j ρ = j ρ ( x ) , sondern mit einem inhomogenen Term, der aus der zweiten Ableitung der Koordinatentransformation aufgebaut ist,

j τ x λ Γ μ v ( x ) λ   =   j ρ x μ j σ x v Γ ρ σ ( j ) τ + 2 j τ x μ x v .

Daher sind alle zweiten Ableitungen Null,

2 j τ x μ x v   =   0 ,

dh die Verwandlung x μ j ρ = j ρ ( x ) ist affin.

Das macht mich unruhig. Kannst du daraus schließen 2 j τ x μ x v = 0 dass die Transformation affin ist? In allgemeinen Mannigfaltigkeiten (wie in der allgemeinen Relativitätstheorie) fehlt die richtige Struktur, um auch nur die affine Transformation zu diskutieren. Tut j μ x μ irgendeine physikalische Bedeutung haben, wenn j μ sind die Koordinaten eines Punktes p und x μ die Koordinaten eines Punktes q ?

Ich hatte das Gefühl, dass ein direkter Beweis nur über die Relation möglich wäre η ich j j ich x p j j x q = η p q , wobei einfache Glattheitseigenschaften der Transformation angenommen und dann einige algebraische Manöver verwendet werden. Ich habe das folgende schöne Argument in dem Buch Gravitation and Cosmology von Steven Weinberg gefunden.

Wir gehen von der Relation aus

η ich j j ich x p j j x q = η p q

Differenzierung bzgl x k wir erhalten

η ich j 2 j ich x p x k j j x q + η ich j j ich x p 2 j j x q x k = 0

Dazu addieren wir die gleiche Gleichung mit p und k vertauscht, und subtrahiere dasselbe mit q und k vertauscht; das ist,

η ich j ( 2 j ich x p x k j j x q + j ich x p 2 j j x q x k + 2 j ich x k x p j j x q + j ich x k 2 j j x q x p 2 j ich x p x q j j x k j ich x p 2 j j x k x q ) = 0

Dies vereinfacht zu

2 η ich j 2 j ich x p x k j j x q = 0

Da die Tensoren j ich x j und η ich j invertierbar sind, bedeutet dies, dass

2 j ich x p x k = 0

Hier möchte ich nur erwähnen, dass es einen direkten Beweis in gibt 1 + 1 Dimensionen mit elementaren Argumenten. Lassen Sie die beiden Patches koordinieren U x und U j (das sind, sagen wir, beide konvexe Mengen in R 2 , die den Ursprung enthalten) haben Lichtkegelkoordinaten x ± und j ± , bzw. Die Metrik lautet

d j + d j   =   d x + d x .

Dies führt zu drei PDEs

j + x + j x +   =   0 j + x +   =   0 Ö r j x +   =   0 ,
j + x j x   =   0 j + x   =   0 Ö r j x   =   0 ,
j + x + j x + j + x j x +   =   1.

Seit det j x 0 , gibt es eigentlich nur zwei Möglichkeiten. Entweder

j x +   =   0   =   j + x ,

oder

j + x +   =   0   =   j x .

Durch eventuelles Umetikettieren x + x , können wir ersteres annehmen. Damit

j +   =   f + ( x + ) a n d j   =   f ( x ) .

Aus dem dritten PDE schließen wir das

f + x + f x   =   1.

Durch Trennung der Variablen ist dies nur möglich, wenn f ± x ± ist unabhängig von x ± . Es folgt dem j ±   =   f ± ( x ± ) sind affine Funktionen. QED

+1 für einen allgemeinen Ansatz, der zB verwendet werden kann, um die Klasse der (infinitesimal) konformen Transformationen abzuleiten.

Nehmen wir zunächst an, dass das erhaltene Skalarprodukt eine positive Signatur hat, um die Hauptidee zu zeigen. Sie sagen auch, Sie möchten keine Homogenität annehmen, aber dies ist bereits in Ihrer Gleichung enthalten, da zur Bildung von Intervallen Unterschiede von Raum-Zeit-Punkten verwendet werden, sodass wir genauso gut einen dieser Punkte als Null eines Vektorraums annehmen können ( entsprechend könnten Sie über die Erhaltung eines Skalarprodukts auf einem Tangentenraum bis zu einem Punkt sprechen, aber dies ist auch linear, nicht affin).

Lassen

f : R 2 R 2 , ( x , j ) ( EIN ( x , j ) , B ( x , j ) )
sei längenerhaltend und nehme an f ist analytisch mit
EIN ( x , j ) = n , m = 0 a n , m n ! m ! x n j m , B ( x , j ) = n , m = 0 b n , m n ! m ! x n j m .

Dann haben wir x 2 + j 2 = EIN ( x , j ) 2 + B ( x , j ) 2 für alle x , j R 2 oder ausdrücklich zuerst

x 2 + j 2 = ( n , m = 0 a n , m n ! m ! x n j m ) 2 + ( n , m = 0 b n , m n ! m ! x n j m ) 2 .
Dies zeigt sofort, dass die einzigen nicht verschwindenden Koeffizienten auftreten, wenn n + m 1 . Wir müssen nur nachforschen n = m = 0 Fall aber das ist trivial da f ( 0 , 0 ) = ( a 0 , 0 , b 0 , 0 ) .

Für die n D-Fall ist die Diskussion völlig analog. Bei willkürlicher Signatur ist Vorsicht geboten, da wir sie nicht verwenden können x 2 + j 2 = 0 x = j = 0 mehr (vielleicht kann man darin arbeiten C anstatt R und analytische Fortsetzung verwenden).

Der letzte verbleibende Bestandteil dieses Arguments ist die Analytizität von f . Aber das ist da trivial | | f ( x , j ) | | 2 = x 2 + j 2 und | | | | 2 sind um jeden herum analytisch ( x , j ) R 2 .

Das OP sagt, er / sie möchte sich nicht auf die Homogenität oder Isotropie des Raums berufen. Indem Sie davon ausgehen, dass der Raum euklidisch ist, nehmen Sie Homogenität und Isotropie an.
@Ben: Ich sollte klarstellen, dass ich nicht davon ausgegangen bin, dass der Raum euklidisch ist. Alles, was ich angenommen habe, war, dass das Skalarprodukt mit positiver Signatur erhalten bleibt, oder allgemeiner, dass jedes Skalarprodukt erhalten bleibt (was alles ist, was OP verlangt). Aber ich werde die Antwort bearbeiten, um dies explizit zu machen.
Und warum ein Downvote? Diese Antwort ist sicherlich richtig, vielleicht mit Ausnahme einiger technischer Details, aber wenn es sich um kleinere Fehler handelt, weisen Sie bitte darauf hin.
OK, ich habe die Ablehnung rückgängig gemacht. Wie in den Kommentaren zu der Frage besprochen, scheint mir, dass das OP eine schlecht definierte oder sich widersprechende Frage gestellt hat, aber ich könnte mich irren.
In Ihrem neuen Text haben Sie "Sie sagen auch, Sie wollen keine Homogenität annehmen, aber dies ist bereits in Ihrer Gleichung enthalten [...]". Das ist mein Punkt. Die Frage erscheint mir unausgereift.
@ Ben: Nachdem ich die Frage noch einmal gelesen habe, stimme ich Ihnen zu. Ich habe gerade bewiesen, dass die Invarianz des Skalarprodukts Linearität impliziert, aber ich frage mich, ob OP dies oder etwas anderes wollte.
Lieber Marek, ist es möglich, etwas mehr Details im Fall Lorentz zu liefern?
@Qmechanic: Ich werde versuchen, später darauf zuzugreifen. Ich habe übrigens festgestellt, dass der euklidische Fall auch mehr Details benötigt (und dann können beide Fälle gleichzeitig behandelt werden).

Lassen Sie uns die Frage von OP wie folgt umformulieren:

Beweisen Sie, dass es eine lokale Koordinatentransformation gibt x μ j ρ = j ρ ( x ) zwischen zwei lokalen Koordinatensystemen (auf einer 3+1-dimensionalen Lorentz-Mannigfaltigkeit) muss affin sein, wenn die Metrik g μ v in beiden Koordinatensystemen zufällig auf konstant flacher Minkowski-Form η μ v .

Hier werden wir einen Beweis präsentieren, der sowohl mit Minkowski- als auch mit euklidischer Signatur funktioniert; in der Tat für jede Signatur und für jede endliche Anzahl von Dimensionen ungleich Null, solange die Metrik g μ v ist invertierbar.

1) Erinnern wir uns zunächst an die Transformationseigenschaft der inversen Metrik g μ v , was eine Kontravariante ist ( 2 , 0 ) symmetrischer Tensor,

j ρ x μ g ( x ) μ v j σ x v   =   g ( j ) ρ σ ,

wo x μ j ρ = j ρ ( x ) ist eine lokale Koordinatentransformation. Denken Sie daran, dass die Metrik g μ v = η μ v ist die flache konstante Metrik in beiden Koordinatensystemen. Damit wir schreiben können

j ρ x μ η μ v j σ x v   =   η ρ σ . ( 1 )

2) Nehmen wir an, dass die lokale Koordinatentransformation reell analytisch ist

j ρ   =   a ( 0 ) ρ + a μ ( 1 ) ρ x μ + 1 2 a μ v ( 2 ) ρ x μ x v + 1 3 ! a μ v λ ( 3 ) ρ x μ x v x λ + .

Durch eventuelles Vornehmen einer entsprechenden Übersetzung gehen wir von nun an ohne Beschränkung der Allgemeinheit davon aus, dass die konstante Verschiebung a ( 0 ) ρ = 0 ist Null.

3) Zur nullten Ordnung in x , Die gleichung ( 1 ) liest

a μ ( 1 ) ρ η μ v a v ( 1 ) σ   =   η ρ σ ,

was nicht überraschend sagt, dass die Matrix a μ ( 1 ) ρ eine Lorentz- (bzw. eine orthogonale) Matrix ist. Durch eventuelles Vornehmen einer entsprechenden "Rotation" gehen wir von nun an ohne Beschränkung der Allgemeinheit davon aus, dass die konstante Matrix

a μ ( 1 ) ρ   =   δ μ ρ

ist die Einheitsmatrix.

4) Im Folgenden ist es zweckmäßig, den Index von zu verringern j σ koordinieren als

j ρ   :=   η ρ σ j σ .

Dann wird die lokale Koordinatentransformation

j ρ   =   η ρ μ x μ + 1 2 a ρ , μ v ( 2 ) x μ x v + 1 3 ! a ρ , μ v λ ( 3 ) x μ x v x λ +
+ 1 n ! a ρ , μ 1 μ n ( n ) x μ 1 x μ n + .

5) Zur ersten Bestellung x , Die gleichung ( 1 ) liest

a ρ , σ μ ( 2 ) + a σ , ρ μ ( 2 )   =   0.

Das ist, a ρ , μ v ( 2 ) ist symmetrisch in μ v , aber antisymmetrisch in ρ μ . Es ist nicht schwer zu sehen (indem man die Symmetrie- und die Antisymmetrieeigenschaft in abwechselnder Reihenfolge jeweils dreimal anwendet), dass die Koeffizienten zweiter Ordnung a ρ , μ v ( 2 ) = 0 muss verschwinden.

6) Zur zweiten Bestellung rein x , Die gleichung ( 1 ) liest

a ρ , σ μ v ( 3 ) + a σ , ρ μ v ( 3 )   =   0.

Das ist, a ρ , μ v λ ( 3 ) ist symmetrisch in μ v λ , aber antisymmetrisch in ρ μ . Für fest λ , können wir wieder zum Schluss kommen a ρ , μ v λ ( 3 ) = 0 .

7) In ähnlicher Weise schließen wir induktiv, dass die Koeffizienten höherer Ordnung a ρ , μ 1 μ n ( n ) = 0 muss auch verschwinden. Damit j μ = x μ . QED

Sehr schön. Ich kam auch zu dem Schluss, dass mein Ansatz eine Induktion in Taylor-Ordnung erfordert (ich habe versucht, einen direkten Weg zu finden, konnte ihn aber nicht finden). Da stellt sich die Frage, ob der direkte Angriff überhaupt möglich ist. Schließlich müssen wir vielleicht die Antisymmetrie ausnutzen, und folglich hängt die polynomiale Transformation entscheidend davon ab, dass wir eine quadratische Form beibehalten und nicht etwa eine kubische oder sogar etwas Inhomogenes. Tatsächlich ist es wahrscheinlich einfach, polynomiale Invarianten zu konstruieren, die durch nicht-polynomiale Transformationen erhalten bleiben.
@Qmechanic: Mir ist der 3. Schritt nicht klar. Insbesondere der Teil, in dem es heißt: "Indem wir möglicherweise eine geeignete "Rotation" durchführen, werden wir von nun an ohne Verlust der Allgemeinheit davon ausgehen, dass die konstante Matrix ..." Können Sie das näher erläutern? Vielen Dank.
@becko: Richtig, ich fasse mich etwas kurz. Ich versuche das auszudrücken, indem ich ein drittes Koordinatensystem einführe j ' , die durch eine standardmäßige lineare Lorentz-Transformation (= eine Art "Rotation") mit der verknüpft ist j Koordinatensystem, können wir davon ausgehen, dass die kombinierte Koordinatentransformation x j ' von der im dritten Schritt beanspruchten Form ist. Die Idee ist, dass wir beweisen können, dass die Koordinatentransformation x j linear ist, wenn wir das irgendwie beweisen können x j ' ist linear. Im Rest des Beweises entfernen wir der Einfachheit halber den Strich aus der Notation. Ist das sinnvoll?
@Qmechanic: Wie beweist man, dass ein Tensor a μ ρ das befriedigt η μ v a μ ρ a v σ = η ρ σ können in das Formular eingetragen werden δ μ ρ durch Lorentztransformationen? Wenn dies elementar sein soll, verweisen Sie mich zumindest auf eine Website oder etwas, das es erklärt, da ich dieses Zeug selbst studiere. Vielen Dank.
@becko: Die Gl. Sie haben gerade geschrieben, ist die abstrakte / beschreibende Definition einer Lorentz-Matrix a μ ρ . Hilft das?
@Qmechanic: Ja, es hilft. Die Frage, die ich in meinem obigen Kommentar gestellt habe, lautet nun: Wie beweist man, dass eine Lorentz-Matrix immer in die Form gebracht werden kann δ μ ρ mit Lorentz-Transformationen? Dies ist gleichbedeutend mit der Frage, ob die Lorentz-Matrizen invertierbar sind, was sie per Definition auch sind. Ich hoffe, ich mache jetzt mehr Sinn.

Die erste Bedingung impliziert, dass die Jacobi-Matrix L j ich = j ich x j ist eine Lorentztransformation. Durch Ersetzen der Definition des Jacobischen in dieser Bedingung erhalten wir:

g ich j j k x ich j l x j = g k l

Insbesondere das Gleichsetzen der Diagonalgleichungen l = k , wir haben

g ich j j k x ich j k x j = g k k = ± 1

(Das Pluszeichen für die Zeitkoordinate und das Minuszeichen für die Raumkoordinaten).

Aber dies ist nur die Hamilton-Jacobi-Gleichung für ein freies relativistisches Teilchen, dessen eindeutige Lösung durch Trennung der Variablen erhalten werden kann:

j k = ich f ich ( k ) ( x ich )

Durch Substitution erhalten wir:

d f ich ( k ) ( x ich ) d x ich = c Ö n s t

Somit sind die neuen Koordinaten lineare Funktionen der alten Koordinaten. Die konstanten Koeffizienten sind nicht unabhängig, da die Jacobi-Matrix eine Lorentz-Transformation sein muss.

Aktualisieren:

Auf Anregung von Lurscher folgen hier zwei Referenzen, die die Hamilton-Jacobi-Gleichung eines relativistischen Teilchens enthalten. (Beide Referenzen beziehen sich auf ein Teilchen in einem äußeren elektromagnetischen Feld. Um die Hamilton-Jacobi-Gleichung für das freie Teilchen zu erhalten, benötigt man den Spezialfall mit verschwindendem Vektorpotential): Referenz-1 (von A. granik) , Referenz- 2

(Die benötigte Version erscheint in Gleichung (33) der ersten Referenz, die zweite Referenz enthält die (richtige) zeitabhängige Version).

Zusätzlich gebe ich hier eine weitere Ableitung basierend auf der WKB-Näherung der Klein-Gordon-Gleichung an:

1 c 2 2 ψ t 2 2 ψ + m 2 C 2 2 ψ = 0

Die Lösungen für ebene Wellen sind gegeben durch:

ψ = C exp ( ich p . x m 2 c 4 + p 2 c 2 t )

Um eine WKB-Approximation durchzuführen, suchen wir eine Lösung der Form:

ψ = EIN ( x , t ) exp ( ich S ( x , t ) )

und nehmen Sie die führenden Terme im Limit 0 . ( S wird manchmal als Hamilton-Jacobi-Phasenfunktion bezeichnet)

Durch Substitution erhalten wir:

( ( 1 c 2 2 EIN t 2 2 EIN ) + 2 ich ( 1 c 2 EIN t S t EIN . S ) EIN 2 ( 1 c 2 2 S t 2 2 S m 2 c 2 ) ) = 0

Der führende Term ist die Hamilton-Jacobi-Gleichung:

1 c 2 2 S t 2 2 S m 2 c 2 = 0

Was als Äquivalent zu jeder Gleichung auf der Hauptdiagonale der in der ursprünglichen Antwort geschriebenen Matrixgleichung angesehen werden kann.

Nun ist auch die Einzigartigkeit der Lösung leicht zu erkennen. Für das freie Teilchen kann man sehen, dass die nicht führenden Terme tatsächlich verschwinden. dh die WKB-Näherung ist exakt.

Die Hamilton-Jacobi-Phasenfunktion S ist nur die Phase der ebenen Wellenlösungen der Klein-Gordon-Gleichung:

S = p . x m 2 c 4 + p 2 c 2 t

An R 4 haben alle Lösungen der freien Klein-Gordon-Gleichung in kartesischen Koordinaten die Form der ebenen Wellen, was impliziert, dass die Hamilton-Jacobi-Phasenfunktion in den kartesischen Koordinaten linear ist.

Können Sie erläutern, wie Sie von der Gleichung, die Sie nach einer Spur erhalten, zu der Aussage gelangen, dass sie einer Hamilton-Jacobi-Gleichung entspricht?
Ich habe ein Update hinzugefügt, das die erforderlichen Ausarbeitungen und Referenzen enthält. Ich habe auch einen Fehler korrigiert: Jeder diagonale Term der Matrixgleichung entspricht einer Hamilton-Jacobi-Gleichung (keine Notwendigkeit, die Spur zu nehmen).
Lieber David Bar Moshe, wie Sie sicher wissen, ist die Lösung nicht eindeutig, wie Sie zu schreiben scheinen (v2). Wir versuchen zu beweisen, dass es affin sein muss. Sie scheinen auch zu behaupten, dass die Lösung notwendigerweise der (additiven) Trennung von Variablen gehorchen muss. Könnten Sie den Grund näher erläutern, vorzugsweise mit einer Referenz?
@Qmechanic Natürlich ist die Lösung nicht eindeutig, da Sie Randbedingungen benötigen, um sie eindeutig zu machen, aber die Lösungsform (Linear in den kartesischen Koordinaten) ist eindeutig. Die Lösung der vier Hamilton-Jacobi-Gleichungen auf der Diagonalen wird mit unterschiedlichen konstanten Koeffizienten erfolgen, die ferner die of-Diagonal-Beziehungen erfüllen müssen, um Matrixkoeffizienten einer Lorentz-Transformation zu sein. Ich habe argumentiert, dass die Hamilton-Jacobi-Phasenfunktion die Phase einer ebenen Wellenlösung der Klein-Gordon-Gleichung ist.
Die Formeindeutigkeit der Lösung wird durch die Linearität der Lösungen der Bewegungsgleichung eines freien Teilchens bestimmt. Trotzdem werde ich versuchen, einen strengeren Beweis für die Eindeutigkeit der Lösung der Variablentrennung zu finden.

Der Beweis entpuppt sich tatsächlich als eine sehr einfache Übung in linearer Algebra. Ich finde diesen algebraischen Beweis sehr zufriedenstellend, da er sehr wenig Maschinen verwendet. Es beweist auch, dass Rotationen und (mit einer leichten Umformulierung) einheitliche Karten linear sind.

Satz: Sei U und v Vektorräume über einem Körper sein F mit bilinearer Form ausgestattet g und h bzw. Weiter davon ausgehen h ist nicht entartet und wir haben eine surjektive Abbildung f : U v so dass h ( f ( u ) , f ( v ) ) = g ( u , v ) für alle u , v v . Dann f ist linear.

Beweis: Let u , v , w v und k F . Dann

h ( f ( k u + v ) k f ( u ) f ( v ) v , f ( w ) ) = h ( f ( k u + v ) , f ( w ) ) k h ( f ( u ) , f ( w ) ) h ( f ( v ) , w ) = g ( k u + v , w ) k g ( u , w ) g ( v , w ) = 0.

Seit w war willkürlich und f ist surjektiv, Nichtentartung von h garantiert das f ( k u + v ) = k f ( u ) + f ( v ) . Deshalb, f ist linear.

Indem U = v = M der dem Minkowski-Raum zugrunde liegende Vektorraum und sein g = h = η als seine Metrik erhalten wir, dass metrierhaltende Transformationen linear sind. Lorentz-Transformationen (distanzerhaltende Transformationen) sind metrische erhaltende Transformationen aufgrund der Polarisationsformel in @Brian Moths Antwort. Ich denke jedoch, dass man die Surjektivität in die Definition einer Lorentz-Transformation einbeziehen muss. Vergleiche mit dem Satz von Mazur-Ulam.

Beachten Sie zunächst, dass if Λ eine Isometrie ist, dann behält sie Skalarprodukte bei, da if p ' = Λ ( p ) , q ' = Λ ( q ) , und r ' = Λ ( r ) dann

( r ' p ' , r ' p ' r ' q ' , r ' q ' q ' p ' , q ' p ' ) / 2 = r ' q ' , q ' p ' ,
und da die LHS erhalten bleibt, muss dies auch die rechte Seite sein.

Beginnen wir mit dem Minkowski-Raum und wählen einen Ursprung aus p , und eine orthonormale Basis e μ befriedigend e μ , e v = η μ v . Jetzt jeder Punkt x im Minkowski-Raum geschrieben werden p + x μ e μ , wo x μ = x p , e μ .

Nun, was ist mit x ' = Λ ( x ) ? Nun, wir wollen natürlich sagen, dass es die gleichen Koordinaten hat. Lassen Sie uns also die neue Basis definieren. e μ ' = Λ ( p + e μ ) Λ ( p ) . Seit Λ bewahrt Produkte von Unterschieden, wir wissen, dass die e μ ' sind orthonormal und so x ' kann geschrieben werden p ' + x ' μ e μ ' , wo x μ ' = x ' p ' , e μ ' .

Aber seit Λ konserviert Produkte, das haben wir x ' μ = x μ . Daher seit

Λ ( p + x μ e μ ) = Λ ( p ) + x μ ( Λ ( p + e μ ) Λ ( p ) ) ,
Λ ist affin.

Hallo @NowIGetToLearnWhatAHeadIs, Sie gehen implizit davon aus, dass die infinitesimale Bedingung d s 2 = d s ' 2 (dh die Bedingung auf dem Tangentialraum) erstreckt sich zu einer endlichen Bedingung s 2 = s ' 2 auf der gesamten Domäne des Diagramms x μ . Dies scheint der schwierige Teil des Beweises zu sein. Kennen Sie irgendein Argument, um zu zeigen, dass dies so sein muss?
(Eigentlich sollte das Standard sein, es bedeutet, dass eine Isometrie zwischen (semi)-riemannschen Mannigfaltigkeiten auch eine Isometrie zwischen (semi)-riemannschen Vektorräumen ist, wenn einer der beiden so ist. Allerdings sieht es für mich nicht ganz trivial aus)
@pppqqq Ich verstehe. Der Beweis erscheint mir auch nicht trivial. Um die Dinge konkret zu machen, betrachte ich nur zwei Kopien des Minkowski-Raums mit einer lokalen Isometrie zwischen ihnen, und ich versuche zu beweisen, dass er linear ist. Mit dem euklidischen Raum ist dies einfach, da gerade Linien die Entfernung minimieren, sodass Linien auf Linien abgebildet werden müssen, aber Sie können diese Tatsache meiner Meinung nach nicht verwenden. So oder so, es gibt wahrscheinlich eine einfache Antwort, die jemand kennt. Soll ich eine neue Frage eröffnen?